Bounded function implies limit is bounded

Click For Summary
The discussion revolves around proving that if a function f is bounded between two values a and b, and the limit of f as x approaches c is L, then L must also lie between a and b. The user is struggling with the proof, particularly with the contradiction method and relating sequences to functions. They express a desire for a clearer understanding of how to formalize their intuitive reasoning about limits and bounded functions. The conversation highlights the challenge of translating informal reasoning into a rigorous mathematical proof. Clarifying these concepts is essential for successfully completing the proof.
miren324
Messages
13
Reaction score
0
Here's the problem:
Let f:D-R and c in R be and a accumulation point of D, which is a subset of R. Suppose that a<=f(x)<=b for all x in D, x not equal to c, and suppose that limx\rightarrowc f(x) = L. Prove that a<=L<=b.

I'm having trouble here. I've tried to prove by contradiction, by assuming that L<a, then after a contradiciton, assuming L>b. This led through a lot of junk and I ended up back where I started. I am unsure how to construct a proof using the analogous relationship of sequences and functions.

Any help would be greatly appreciated.
 
Physics news on Phys.org
What is your intuition? If you were going to explain informally the reason, what would you say?
 
It just makes sense that it would be true. I know that if x does not equal c, the limit as x approaches c of f(x) is L, and L is "near" f(c-.00001) and f(c+.00001) and f(c-.000000001), etc. But I only know that from experience with linear equations. I'm not sure how to translate this into a formal proof for all functions, domains, and limits.
 
Question: A clock's minute hand has length 4 and its hour hand has length 3. What is the distance between the tips at the moment when it is increasing most rapidly?(Putnam Exam Question) Answer: Making assumption that both the hands moves at constant angular velocities, the answer is ## \sqrt{7} .## But don't you think this assumption is somewhat doubtful and wrong?

Similar threads

  • · Replies 11 ·
Replies
11
Views
2K
  • · Replies 2 ·
Replies
2
Views
2K
Replies
4
Views
2K
  • · Replies 13 ·
Replies
13
Views
2K
  • · Replies 6 ·
Replies
6
Views
2K
  • · Replies 14 ·
Replies
14
Views
2K
Replies
3
Views
1K
  • · Replies 2 ·
Replies
2
Views
1K
  • · Replies 3 ·
Replies
3
Views
2K
  • · Replies 8 ·
Replies
8
Views
1K